Given a quadratic equation in vertex form, find the vertex, axis of symmetry, whether the graph
opens up or down, the maximum or minimum, and the y-intercept. Graph it!
16. y = -2(x + 2)² +4
Vertex:
Axis of symmetry:
Opens: up
Maximum
down
Minimum
X=h A must write
x=
Max/Min Value:

Answers

Answer 1

The key features of the given quadratic equation include the following:

Vertex = (-2, 4)

Axis of symmetry: x = -2.

Direction of opening: downward.

There is a maximum (max) at 4.

y-intercept = (0, 0).

roots: x = -4 and x = 0.

What is the vertex form of a quadratic equation?

In Mathematics and Geometry, the vertex form of a quadratic equation is modeled by this formula:

f(x) = a(x - h)² + k

Where:

h and k represents the vertex of the graph.a represents the leading coefficient.

Based on the information provided about the graph of this quadratic equation, we can reasonably infer and logically deduce that its vertex can be determined as follows:

y = -2(x + 2)² +4

Vertex, (h, k) = (-2, 4)

Axis of symmetry, Xmax = -2.

Read more on vertex here: brainly.com/question/14946018

#SPJ1

Given A Quadratic Equation In Vertex Form, Find The Vertex, Axis Of Symmetry, Whether The Graphopens

Related Questions

Suppose On a Sunny Day the Temperature decreases 5.4 F° for each 1,000- foot rise in elevation. If the temperature at the base of a 3,000 foot mountain is 27 F°, what is the temp at the mountain summit?

GUYS PLS EXPLAINNN

Answers

Therefore, the temperature at the summit of the 3,000-foot mountain on a sunny day is estimated to be 10.8 F°.

What is equation?

In mathematics, an equation is a statement that indicates the equality of two expressions. An equation typically contains one or more variables, which are placeholders for unknown values or quantities. The variables can take on different values, and the goal is often to find the values that satisfy the equation.

Here,

Let's start by calculating the rate of change of temperature with respect to elevation:

=-5.4 F° / 1,000 ft

This means that for every 1,000-foot increase in elevation, the temperature will decrease by 5.4 F°.

Next, we can calculate how much the temperature will decrease from the base of the mountain to the summit:

3,000 ft / 1,000 ft = 3

This means that the elevation difference between the base of the mountain and the summit is 3,000 - 0 = 3,000 ft.

So, the temperature decrease from the base to the summit is:

-5.4 F° / 1,000 ft * 3,000 ft = -16.2 F°

This means that the temperature at the summit will be:

27 F° - 16.2 F° = 10.8 F°

To know more about equation,

https://brainly.com/question/28243079

#SPJ1

refer to exercise 3.91. in this exercise, we determined that the mean and variance of the costs necessary to find three employees with positive indications of asbestos poisoning were and , respectively. do you think it is highly unlikely that the cost of completing the tests will exceed ?

Answers

The probability that the cost will exceed $600 is approximately 0.057, or 5.7%. which is relatively small, so it is highly unlikely that the cost of conducting the test will exceed $600.

we need to use the information provided to calculate the probability that the test execution cost will exceed a certain value.

Let X be the cost of conducting a test to find three of her employees with positive signs of asbestos poisoning. From the information provided, we can see that:

E(X) = $550

Var(X) = $500

Using these values, we can standardize X to a standard normal distribution.

Z = (X - E(X)) / sqrt(Var(X))

= (X - $550) / square meters ($500)

Accepting that the costs take after an ordinary conveyance, able to utilize the standard ordinary conveyance to compute the likelihood that the fetched surpasses a certain esteem. For case, to find the likelihood that the taken toll surpasses $600, we are able to compute:

P(X > $600) = P(Z > ($600 - $550) / square meters ($500))

= P(Z > 1.58)

Using a standard normal table or calculator, we find that P(Z > 1.58) is approximately 0.057. hence, the probability that the cost will exceed $600 is approximately 0.057, or 5.7%.

This likelihood is generally little, so it is profoundly impossible that the fetch of conducting the test will surpass $600.

Be that as it may, the precise limit for what is considered exceedingly improbable may change depending on the setting and the particular criteria utilized. 

learn more about probability

brainly.com/question/30034780

#SPJ4

A population has a mean μ=135 and a standard deviation Ï=28. Find the mean and standard deviation of the sampling distribution of sample means with sample size n=57.

Answers

A population has a mean μ=135 and a standard deviation Ï=28.

The mean of the sampling distribution of sample means is equal to the population mean, that is 135.

The standard deviation of the sampling distribution of sample means can be calculated by using the formula

SE = Ï /[tex]\sqrt{n}[/tex]

Where Ï is the population standard deviation and n is the sample size.

By putting the given values we get

SE = 28 / [tex]\sqrt{57}[/tex] = 3.7

Hence, the mean of the sampling distribution of sample means is 135, and the standard deviation is 3.7.

To know more about standard deviation here

https://brainly.com/question/28075377
#SPJ4

dont understand what m supposed to do at the radius = diameter/2 part

Answers

Step-by-step explanation:

Radius simply means half of the diameter. In other words radius is the distance between the center of the circle to the outside of the circle

1/4 exponent 4 equal to in fraction form

Answers

Answer:

(1/256)

Step-by-step explanation:

[tex](\frac{1}{4})^{4} = \frac{1^{4} }{4^{4} } = \frac{1}{256}[/tex]

Using laws of exponents I distributed the exponent to the fraction and solved.

please solve and explain

Answers

LaTisha's calculation of the distance for the brake anchor placement is incorrect because she didn't take into account the height at which the cable is attached to the starting tree.

What is the correct distance ?

The bungee cord is attached 16 feet above the ground, which will also contribute to the length of the cable needed to reach the ending tree.

To correctly calculate the distance, we can use the Pythagorean theorem, considering the height (16 feet) and the length of the stretched bungee cord (24 feet + 18 feet = 42 feet). Let's denote the horizontal distance from the base of the ending tree to the brake anchor as 'd'. We have:

d² + 16² = 42²

Now, we can solve for d:

d² + 256 = 1764

d² = 1508

d = √1508 = 38.8 feet (rounded to one decimal place)

The brake anchor should be placed approximately 38.8 feet from the base of the ending tree.

Find out more on distance at https://brainly.com/question/27672464

#SPJ1

Think of a number. The L C M of this number and 42 is 126. If the number lies between 60 and 70, what is the number

Answers

The LCM of 63 and 42 is 126 where 63 is the required number lying in between 60 and 70.

Let the missing number be x.

The number x lies in between 60 and 70.

The LCM ( Least Common Multiple) refers to the least value that is divisible by any two (or more) numbers.

Here LCM of x and 42 is 126.

Simplifying 126 we can write it as,

126 = 2*3*21

Thus, one of the number having 126 is as multiple is 42  (as already given).

The other number, that is x, having 126 as multiple lying between 60 and 70 is,

x = 3*21 = 63

To know more about Least Common Multiple here

https://brainly.com/question/20380514

#SPJ4

What is the surface area of this composite solid? Show your work.

Answers

According to the above, we have to add the surface area of each face. So, the surface area of this solid would be 127.

What is the surface area of the composite solid?

To calculate the surface area of the composite solid we have to perform the following procedure:

Sides (a) area

4 * 3 = 1212 * 2 = 24

Sides (b) area

7 * 3 = 2121 * 3 = 63

Face (c) area

3 * 4 / 2 = 66 * 2 = 12

Base area

4 * 7 = 28

Total surface area

To calculate the total surface area we have to add the value of each face as shown below:

28 + 12 + 63 + 24 = 127

According to the above, the surface area of this solid would be 127.

Learn more about surface area in: https://brainly.com/question/29298005

#SPJ1

The graph of EFG is shown. Graph the image of EFG after translation of 3 units left 1 unit down write the coordinates of the image

Answers

The coordinates for the image after the translation of EFG would be E'(-2, 3), F'(-4, 0), and G'(-1, -2).

How to translate an image ?

To translate a point, you simply add or subtract the specified units from its x and y coordinates. In this case, you are asked to translate the points 3 units left and 1 unit down.

To translate 3 units left, subtract 3 from the x-coordinate, and to translate 1 unit down, subtract 1 from the y-coordinate.

For point E (1, 4):

E' = (1 - 3, 4 - 1) = (-2, 3)

For point F (-1, 1):

F' = (-1 - 3, 1 - 1) = (-4, 0)

For point G (2, -1):

G' = (2 - 3, -1 - 1) = (-1, -2)

The translated points are E'(-2, 3), F'(-4, 0), and G'(-1, -2).

Find out more on image translation at https://brainly.com/question/31019824

#SPJ1  

WHAT IS THE RANGE OF THIS PIECEWISE FUNCTION?

Answers

The given piecewise function has a range of:

Range = {46, 48, 50, 52, 54, 56}.

What is range of a function?

The range of a function is the set of all possible output values, or the set of all y-coordinates that correspond to the x-coordinates in the domain of the function.

It is, in other words, the entire set of values that the function is capable of returning as its result.

From the given piecewise function, we can see that the y-coordinates of the function are limited to the values 46, 48, 50, 52, 54, and 56.

These values correspond to the y-intercepts of each of the line segments in the graph.

To know  more about piecewise function visit:

https://brainly.com/question/29975410

#SPJ1

Given: Circle with center D.
Construct: Equilateral triangle ABC so that points A, B, and C are on circle D.​

Answers

The circle with equilateral triangle is constructed.

What is triangle?

A triangle is a form of polygon with three sides; the intersection of the two longest sides is known as the triangle's vertex. There is an angle created between two sides. One of the crucial elements of geometry is this.

Here we need to construct the circle  with center D.

Now we know that in equilateral triangle , side length of the all sides are equal.

Then, AB=BC=CA.

To learn more about triangle refer the below link

https://brainly.com/question/17335144

#SPJ1

Q3. (Calculator)

Abox contains only red, blue and green pens.

The ratio of red pens to blue pens is 5:9.

The ratio of blue pens to green pens is 1:4.

Calculate the percentage of pens that are blue.

Answers

The percentage of pens that are blue =18%

Let's assume that r represents the red pens, b represents the blue pens and g represents the green pens.

From the statements we can observe that the blue appears in both ratio.

The ratios are,

r:b=5:9 and b:g=1:4

Consider ratio b:g,

b:g

= 1:4

= (1×9) : (4×9)

=9:36

i.e., b:g = 9:36 and r:b = 5:9

So, we can conclude that, b=9, g=36, r=5

so, the total number of pens would be,

n = b + g + r

n = 9 + 36 + 5

n = 50

Now we need to find the percentage of pens that are blue.

Using percentage formula,

p = (b/n) × 100

p = (9/50) × 100

p = 18%

Hence, the percentage of blue pens are 18%.

Learn more about the percentage here:

https://brainly.com/question/16797504

#SPJ4

A rectangle is seven times as long as it’s width. One way to write an expression to find the perimeter would be m+7m+m+7m. Write the expression in two other ways.

Answers

Step-by-step explanation:

Perimeter =  2 m + 2 * 7m

Perimeter = 2 ( m + 7m)

Perimeter = 2m + 14m

Perimeter = 16 m                          Pick any of them

Is 409  greater or less than 1.008

Answers

409 Is greater than 1.008

Answer:

409 > 1.008

Step-by-step explanation:

409 is greater than 1.008

9. A classroom table has a mass of 24 kg. Pulling with a force of 36 N, a student moves the table to the other side of the classroom. How much did the table accelerate? Record your answer and fill in the bubbles. Be sure to use the correct place value. ​

Answers

The acceleration of this classroom table is equal to 1.5 meter per seconds square.

How to calculate the acceleration of this classroom table?

From Newton's Second Law of Motion, the force exerted on the classroom table is modeled or represented by this mathematical equation (formula):

F = ma

Where:

F represent the force.m represent the mass.a represent the acceleration.

By making "acceleration" the subject of formula, we have the following:

Acceleration, a = force/mass

Acceleration, a = 36/24

Acceleration, a = 1.5 meter per seconds square.

Read more on mass here: brainly.com/question/14344386

#SPJ1

can someone js help me w these questions

Answers

A:60, because 12x5
B:7, because 45 divided by 9 = 5+2
C:62, because 100-38 is 63
D:25, because 11x5 is 55 than 55-30=25

Answer:

a=60 (12*5=60)

b=7 (45/9=5 5+2=7)

c=62

d=25 (11*5=55 55-30=25)

A. Find the average rate of change over the interval (-1, 2] for each function. Show all steps and work for credit.

B. Which function has the greatest average rate of change over the interval [1, 2]

Answers

the average rate of change over the interval (-1, 2] in the given function is 1.17.all three functions have the same average rate of change over the interval [1, 2], which is 2

what is function and average rate?

A function is a mathematical rule that takes an input and produces an output. The average rate of change of a function over an interval is the average amount the output changes per unit change in the input over that interval.

According to given information

For the function f(x) = 2x + 3:

Let x1 = -1 and x2 = 2

f(x1) = 2(-1) + 3 = 1

f(x2) = 2(2) + 3 = 7

The average rate of change is:

[f(x2) - f(x1)] / [x2 - x1] = [7 - 1] / [2 - (-1)] = 6 / 3 = 2

Therefore, the average rate of change of f(x) over the interval (-1, 2] is 2.

For the function g(x) = x^2 - 1:

Let x1 = -1 and x2 = 2

g(x1) = (-1)^2 - 1 = 0

g(x2) = 2^2 - 1 = 3

The average rate of change is:

[g(x2) - g(x1)] / [x2 - x1] = [3 - 0] / [2 - (-1)] = 3 / 3 = 1

Therefore, the average rate of change of g(x) over the interval (-1, 2] is 1.

For the function h(x) = 2^x + 1:

Let x1 = -1 and x2 = 2

h(x1) = 2^(-1) + 1 = 1.5

h(x2) = 2^2 + 1 = 5

The average rate of change is:

[h(x2) - h(x1)] / [x2 - x1] = [5 - 1.5] / [2 - (-1)] = 3.5 / 3 = 1.17 (rounded to 2 decimal places)

Therefore, the average rate of change of h(x) over the interval (-1, 2] is approximately 1.17.

b)To determine which function has the greatest average rate of change over the interval [1, 2], we need to calculate the average rate of change for each function over that interval and compare the results.

Let's assume the functions are f(x), g(x), and h(x).

The average rate of change for f(x) over the interval [1, 2] is:

[f(2) - f(1)] / [2 - 1] = (2(2) + 3 - 2(1) - 3) / 1 = 2

The average rate of change for g(x) over the interval [1, 2] is:

[g(2) - g(1)] / [2 - 1] = (2^2 - 1 - 1^2 + 1) / 1 = 2

The average rate of change for h(x) over the interval [1, 2] is:

[h(2) - h(1)] / [2 - 1] = (2^2 + 1 - 2^1 - 1) / 1 = 2

Therefore, all three functions have the same average rate of change over the interval [1, 2], which is 2.

To know more about Function Visit:

https://brainly.com/question/14418346

#SPJ1

A piece of metal with a mass of 611 g is placed into a graduated cylinder that contains25.1 mL of water, raising the water level to 56.7 mL. What is the density of the metal?A) 2.70 g/cm3 B) 7.13 g/cm3 C) 8.96 g/cm3 D) 10.5 g/cm3 E) 19.3g/cm3

Answers

The density of the given metal is 19.3g/cm³ . Then, the correct answer to this given question is Option E.

The given mass of  the metal is 611 g  and it is kept in a elevated cylinder that contains 25.1 mL of water,
Now, let us increase the water level to 56.7 mL. The metal  volume can be evaluated by subtracting the initial volume  from the final volume of water released by the metal.

Hence,
Metal volume = final volume - initial volume
= 56.7 mL - 25.1 mL
= 31.6 mL

Therefore,
Density = Mass / Volume
= 611 g/31.6 mL
= 19.3 g/cm³

The density of the given metal is 19.3g/cm³ . Then, the correct answer to this given question is Option E.


To learn more about density,
https://brainly.com/question/1354972
#SPJ4

please help asap!! i need the help

Answers

Answer:

Step-by-step explanation:

angle L is 106 degrees

x= 2

angle LMK is 37 degrees

its equal, you can check by adding up all the angles to 360 degrees

put on pair of brackets into each calculation to make it correct

a. 6×7-5 +4= 16

b. -2[tex]x^{2}[/tex]+24÷12-4=2

Answers

(a) The simplified and correct expression is (6×7)-(5 +4) = 16

(b) The simplified and correct expression is (-2x²) + (24÷12)-4 = 2

What is the simplified expression?

a. 6×7-5 +4= 16

In this calculation, we need to use brackets to ensure the correct order of operations, which is typically remembered using the acronym PEMDAS (Parentheses, Exponents, Multiplication and Division [from left to right], Addition and Subtraction [from left to right]).

Without brackets, the calculation would be evaluated as follows:

6×7-5 +4 = 42-5 +4 (applying multiplication first)

= 37 +4 (applying subtraction)

= 41 (applying addition)

However, the desired result is 16. To achieve this, we can use brackets to group the addition and subtraction operations together, like this:

(6×7)-(5 +4) = 42 - 9 (applying addition within the brackets first)

= 33 (applying subtraction)

b. -2x²+24÷12-4=2

Similarly, in this calculation, we need to use brackets to ensure the correct order of operations.

Without brackets, the calculation would be evaluated as follows:

-2x²+24÷12-4 = -2x²+2-4 (applying division first)

= -2x²-2 (applying addition and subtraction)

However, the desired result is 2. To achieve this, we can use brackets to group the division and subtraction operations together, like this:

-2x²+(24÷12)-4 = -2x²+2-4 (applying division within the brackets first)

= -2x²-2 (applying addition and subtraction)

By adding brackets to each calculation to ensure the desired order of operations, we arrive at the correct results of 16 and 2, respectively.

Learn more about PEMDAS here: https://brainly.com/question/11421179

#SPJ1

If Joanne has three hours and 45 minutes before swim practice, she wants to go to the store which will take 220 minutes. She also wants to take her brother to the park for 230 minutes which activity does Joanne have time to do before practice

Answers

3. hrs 45 mins = 225 minutes so the answer is
Go to the store

Dave is buying popcorn and sodas for his son and his five friends that he brings to the movies (Six kids total). The needs to buy at least one of the two items for each of the six. Popcorn costs $2. 50 per bag and sodas cost $4. 00 each. Dave can spend at most $40. Lets represent the number of sodas he buys and p represent the number of bags of popcorn he buys

Answers

The system of equations that models this scenario is:

p + s ≥ 62.50p + 4.00s ≤ 40

How can it be modeled using a system of equations?

Let p be the number of bags of popcorn.

Let s be the number of sodas Dave buys.

Since Dave needs to buy at least one of each item for all six kids, we have:

p + s = 6 -----(1)

Given:

The cost of a bag of popcorn is $2.50.

The cost of a soda is $4.00.

The total cost of Dave's purchase can be expressed as:

2.50p + 4.00s = C -----(2) where the C is the total cost of Dave's purchase.

Since Dave can spend at most $40, we have:

2.50p + 4.00s ≤ 40 -----(3).

Read more about system of equations

brainly.com/question/13729904

#SPJ4

At a company a copy machine prints 175 pages in 5 minutes. if the number of pages printed is proportional to the time in minutes what is the unite rate

Answers

Answer:

The unit rate is 35 pages per minute.

Step-by-step explanation:

Divide 175 by 5, or 175/5  to get the unit rate, which is 35.

Change the subject of each formula to the letter given in brackets.

Answers

Here he formula for v is:

v = √(2gh)

What is meant by the term formula?

A formula is a mathematical relationship or rule that is expressed using symbols and mathematical operations. It is used to represent a relationship between quantities or to calculate a value based on given variables or inputs. Formulas are often used in various branches of mathematics, science, and engineering.

According to the given information

To change the subject of the formula mgh = (1/2)mv² to v, we need to isolate v on one side of the equation.

First, we can multiply both sides of the equation by 2 to eliminate the fraction:

2mgh = mv²

Next, we can divide both sides of the equation by m:

(2mgh) / m = v²

Simplifying the left side, we get:

2gh = v²

Finally, we can take the square root of both sides of the equation to solve for v:

v = √(2gh)

Therefore, the formula for v is:

v = √(2gh)

To know more about formula visit

brainly.com/question/30098455

#SPJ1

PLEASE HELP!!!

A principal of $4200 is invested at %8 interest, compounded annually. How much will the investment be worth after 12 years?

Answers

Answer:

$10,244.14 after 12 years

Step-by-step explanation:

To solve this problem, we can use the formula for compound interest:

A = P(1 + r/n)^(n*t)

A = the final amount of the investment

P = the principal amount ($4200 in this case)

r = the annual interest rate (8% or 0.08 as a decimal)

n = the number of times the interest is compounded per year (once annually in this case)

t = the number of years the money is invested (12 years in this case)

Substituting the given values into the formula, we get:

A = 4200(1 + 0.08/1)^(1*12)

A = 4200(1.08)^12

A = 4200(2.44140625)

A = $10,244.14 (rounded to the nearest cent)

the average score of 100 students taking a statistics final was 70, with a standard deviation of 7. assuming a normal distribution, what test score value separates the top 2.5% of the students from the rest of the students? (show your work)

Answers

The average score of 100 students taking a statistics final was 70, with a standard deviation of 7. The test score esteem that isolates the beat 2.5% of normal distribution from the rest of the understudies is roughly 83.72.

To discover the test score esteem that isolates the best 2.5% of the understudies, we got to discover the z-score comparing to that rate utilizing the standard ordinary conveyance table.

z = (x - μ) / σ

To discover the z-score compared to the best 2.5%, we see up the region of the right-hand tail of the standard normal distribution table, which is 0.025. This compares to a z-score of roughly 1.96.

Presently ready to utilize the z-score equation to unravel for x:

1.96 = (x - 70) / 7 Increasing both sides by 7, we get:

x - 70 = 13.72

Including 70 to both sides, we get:

x = 83.72

Hence, the test score esteem that isolates the beat 2.5% of the understudies from the rest of the understudies is roughly 83.72.

 

To learn about normal distribution visit:

https://brainly.com/question/29509087

#SPJ4

please help me IXL have to achieve a smart score of 80 part 1

Answers

The equation is linear and can be written in the slope intercept form as: y = -3x/5 + 6.

What is a linear equation

A linear equation is an algebraic equation that represents a straight line on a graph. It can be written in the form y = mx + b, where x and y are variables, m is the slope of the line, and b is the y-intercept, which is the point where the line crosses the y-axis.

given the slope m = -3/5 and y intercept b = 6, we can write the equation as;

y = (-3/5)x + 6

y = -3x/5 + 6.

Therefore, the equation is linear and can be written in the slope intercept form as: y = -3x/5 + 6.

Read more about equation here:https://brainly.com/question/4074386

#SPJ1

Two real numbers are defined as: a=0444444444444 b=0.354355435554 Determine whether each number is rational or irrational. Is the product of a and b rational or irrational? Justify your answers. Enter your answers and your justifications in the box provided.​

Answers

The a is rational , b is irrational and product of a and b is rational.

The number a=0.444444444444 is rational

Because it can be expressed as the ratio of two integers.

a = 4/9

The number b=0.354355435554 is irrational

Because it cannot be expressed as the ratio of two integers.

It has an infinite non-repeating decimal expansion.

To determine whether the product of a and b is rational or irrational, we can simply multiply them together:

a × b = (4/9) × 0.354355435554

= 0.158919191574

This product is a rational number, because it can be expressed as the ratio of two integers.

Hence, a is rational , b is irrational and product of a and b is rational.

To learn more on Number system click:

https://brainly.com/question/22046046

#SPJ1

PLEASE HELP ME ITS URGENT MY GRADE NEEDS HELP! THIS IS MULTIPLE CHOICE QUESTION!

Which ordered pairs are solutions to the inequality 2x+3y≥−1?


Select each correct answer.

Responses


(0, −1)

begin ordered pair 5 comma negative 1 end ordered pair


(−2, 1)

begin ordered pair negative 2 comma 1 end ordered pair


(0, 1)

begin ordered pair 0 comma 1 end ordered pair


(−6, 0)

begin ordered pair negative 6 comma 0 end ordered pair


(2, −1)

Answers

Answer:

b, c, d, f

Step-by-step explanation:

Which ordered pairs are solutions to the inequality?

 2x+3y≥−1?

a≥b means that A must be equal to or greater than B.

Let's do this the long, but easy way, by plugging it in!

(x,y)

a. (0,-1) -> -3≥-1 -> false

b. (5,-1)-> 7≥-1 -> true

c. (-2,1) -> -1≥-1 -> true

d. (0,1) -> 3≥-1 -> true

e. (-6,0) -> -12≥-1 - > false

f. (2,-1) -> 1≥-1 -> true

Let me know if it is incorrect!

- a friendly 8th grader :)

What is the equation of the trend line in
the scatter plot?

Use the two yellow points to write the
equation in slope-intercept form. Write
any coefficients as integers, proper
fractions, or improper fractions in
simplest form.

Answers

Answer:

y= 7/3x -7

Step-by-step explanation:

Slope is 7/3 (rise/run or up seven over 3) and when you continue the graph the line will eventually hit your y-intercept at -7

Other Questions
A(n) _________ is always associated with an exact instruction in pipelined computers.precise interruptimprecise interrupt An amusement park has three roller coasters, the "Falcon Flyer placed at (50, 300), the "Sprinter" placed at (400, 550), and the "Air Rider" placed at (800, 250). All measurements are in meters. You are at a point half way between the "Falcon Flyer" and the "Sprinter", see a long line, and decide to take a shortcut to the "Air Rider". How far will you walk on this shortcut if it follows a straight path? Round your answer to the nearest meter Why did Scout and Jem not leave the school until almost everyone else had gone? How did the Porfiriato work to attract the FDI? (Foreign Direct Investment) what would go in the red square? (t / f) based on their innovative nature, startups are less vulnerable to competition than established companies Question 80The term "half-value layer" (HVL) is used to designate the thickness of a particular material that will reduce, by one-half, the intensity of radiation passing through the material. Beta radiation is commonly eliminated bya. Leadb. Concretec. Glass or plasticd. aluminum During his time in prison, Sam worked in the cafeteria as much as he was allowed. He volunteered to run weekly writing classes for inmates who were interested. Even though Sam is sentenced to 30 years in prison, he will be eligible for early release if he is deemed rehabilitated. What is the term that describes this early release?probationparolerehabilitationdeath row The following causes a data hazard for the 5-stage MIPS pipeline i1: add $s0, $t0, $t1 i2: sub $t2, $s0, $t3 "Forwarding" can resolve the hazard by providing the ALU's output (for i1's stage 3) directly to the ALU's input (for i2's stage 3).YesNo In a high-performance router, shadow copies of the routing table are kept in I. the input ports II. the output ports III. the switching fabric IV. all of the above Both psychodynamic theorists and cognitive-behavioral theorists believe that in conversion disorder, the symptoms that appear: Read the excerpt from Beowulf.Then a proud warriorquestioned the men concerning their origins:Where do you come from, carrying thesedecorated shields and shirts of mail,these (Can't Say Word) helmets and javelins?I am Hrothgars herald and officer.I have never seen so impressive or largean assembly of strangers. Stoutness of heart,bravery not banishment, must have brought you to Hrothgar.Based on this excerpt, the reader can infer that the speaker is _________ the strangers.A. In Awe Of.B. Disdainful Of.C. Frightened By.D. Disinterested In. Suppose C = 700, 1 = 200, G = 100, and X=0. What is the gross domestic product (GDP)? Calculate each component's share of GDP. GDP = C's share = % I's share = % G's share = % X's share = % identifying subjects and verb What do you want for dinner A strategic plan addresses an organization's plan for which key functional areas? (Check all that apply.)a. Marketingb. Human resourcesc. Financed. Risk management Which best describes the tempo of the music in the first three sections (Introduction, Dance of the Youths and Maidens, and Game of Abduction) of Stravinsky's The Rite of Spring? If 6.00 moles of CaO is combined with CO2, how many grams of CaCO3 would be formed How did Mbuyiseni Mtshali in Always a suspect poem manage to to bring across the message of prejudice, fear and oppression karly is introducing her friend, the photographer willy klein. she spends a few minutes telling the audience how she and willy met, barely mentioning his past work or his new series of door photographs, about which he will be speaking. many in the audience are confused about who willy klein is, and the applause is unenthusiastic when he takes the stage. what errors did karly make? You rolled a number cube 20 times, you get three 14 times. What is the experimental probability that you will roll a three on your next roll?